It can be inferred from the passage that the author would be most likely to regard a policy in which all forest fires...

js9212 on November 10, 2019

Why is A wrong

Can you explain why A is not the answer?

Reply
Create a free account to read and take part in forum discussions.

Already have an account? log in

Irina on November 10, 2019

@jd9212,

The issue with (A) is the author would likely regard a policy that would allow ALL fires started by lightening as too extreme. The author tells us that in certain circumstances it could a fire started by lightning could lead to total devastation (lines 20-24) rather that serve as means of restoring forests as (A) suggests. The author then argues that such fires could be part of wildfire management system that involves selective harvesting and prescribed fires, but only under the right conditions "when the weather is dump enough to reduce the risk of extensive damage" (lines 50-54).